2012 USAJMO Problems/Problem 5
Contents
Problem
For distinct positive integers , , define to be the number of integers with such that the remainder when divided by 2012 is greater than that of divided by 2012. Let be the minimum value of , where and range over all pairs of distinct positive integers less than 2012. Determine .
Solutions
Solution 1
First we'll show that , then we'll find an example that have .
Let be the remainder when is divided by 2012, and let be defined similarly for . First, we know that, if and $bk \nequiv 0 \pmod 2012$ (Error compiling LaTeX. Unknown error_msg), then and . This implies that, since and , . Similarly, if and $ak \nequiv 0 \pmod 2012$ (Error compiling LaTeX. Unknown error_msg) then , establishing a one-to-one correspondence between the number of such that . Thus, if is the number of such that and , then . Now I'll show that .
If , then I'll show you that . This is actually pretty clear; assume that's not true and set up a congruence relation: Since is relatively prime to 2012, it is invertible mod 2012, so we must have . Since 0 < a, b <2012a=bx_k \neq y_kgcd(k, 2012)=1x_k \neq 0 \neq y_knnn \geq \phi(2012) = \phi(503*4) = 1004S \geq 502$.
To show 502 works, consider$ (Error compiling LaTeX. Unknown error_msg)(a, b)=(1006, 2)kx_k=0f(1006, 2)k = 503, 503*3x_k = y_k = 1006f(1006, 2)x_k \neq 0 \neq y_k2012-kx_k > y_kx_{2012-k} > y_{2012-k}f(1006, 2)\frac{1004}{2} = 502 \geq f(1006, 2) \geq S \geq 502S=502$. === Solution 2 ===
Let$ (Error compiling LaTeX. Unknown error_msg)ak \equiv r_{a} \pmod{2012}bk \equiv r_{b} \pmod{2012}a(2012 - k) \equiv 2012 - r_{a} \pmod{2012}b(2012 - k) \equiv 2012 - r_{b} \pmod{2012}kr_{a} > r_{b}kr_{b} > r_{a}\frac{1}{2}kr_{a} \neq r_{b}r_{a} \neq 0$.
However, the answer is NOT$ (Error compiling LaTeX. Unknown error_msg)\frac{1}{2}(2012) = 1006kr_{a} = r_{b}r_{a} = 0$.
So, let's start counting.
If$ (Error compiling LaTeX. Unknown error_msg)ka \equiv 0 \pmod{1006}a - b \equiv 0 \pmod{1006}a = 1006a = b + 10061005kkk$at all).
If$ (Error compiling LaTeX. Unknown error_msg)kk = 503k = 503 \cdot 3a \equiv 0 \pmod{4}a \equiv b \pmod{4}ak \equiv 0 \pmod{2012}ak \equiv bk \pmod{2012}a, b < 20122k$.
In total, we have$ (Error compiling LaTeX. Unknown error_msg)2 + 1005 = 1007kr_{a} = r_{b}r_{a} = 02011 - 1007 = 1004kr_{a} \neq r_{b}r_{a} \neq 0\frac{1}{2} \cdot 1004 = \boxed{502}$.
Solution by$ (Error compiling LaTeX. Unknown error_msg)\textbf{\underline{Invoker}}akbka(2012-k) b(2012-k)2 \pmod{4}2012=2^2*5035035032\pmod{4}21006502$.
--[[User:Va2010|Va2010]] 11:12, 28 April 2012 (EDT)va2010
=== Solution 4 === Say that the problem is a race track with$ (Error compiling LaTeX. Unknown error_msg)20122012=2^2*503503$.
See also
2012 USAJMO (Problems • Resources) | ||
Preceded by Problem 4 |
Followed by Problem 6 | |
1 • 2 • 3 • 4 • 5 • 6 | ||
All USAJMO Problems and Solutions |
The problems on this page are copyrighted by the Mathematical Association of America's American Mathematics Competitions.